You are on page 1of 9

PHY4604 Fall 2007 Problem Set 5 Solutions

PHY 4604 Problem Set #5 Solutions


Problem 1 (20 points): Use separation of variables in Cartesian coordinates to solve the infinite
cubical well (or “particle in a box”):
⎧ 0 0 < x < L,0 < y < L,0 < z < L
V ( x, y , z ) = ⎨
⎩+ ∞ otherwise

(a) (10 points) Find the stationary states and the corresponding energies.
8 h 2π 2 2
Answer: ψ nxny nz ( x, y, z ) = 3
sin( n xπ x / L ) sin( n y πy / L ) sin( n z πz / L ) and E = 2
(nx + n y2 + nz2 )
L 2mL
Solution: In this case the V(x,y,z) = V(x)+V(y)+V(z) and we look for stationary state solutions
of the form ψ ( x, y, z ) = X ( x)Y ( y ) Z ( z ) and Schrödinger’s equation becomes
1 ⎛ h2 d 2 X ⎞ 1 ⎛ h 2 d 2Y ⎞ 1 ⎛ h2 d 2Z ⎞
⎜⎜ − 2
+ V ( x ) X ⎟
⎟ + ⎜
⎜ − 2
+ V ( y )Y ⎟⎟ + ⎜⎜ − 2
+ V ( z ) Z ⎟⎟ = E
X ⎝ 2 m dx ⎠ Y ⎝ 2m dy ⎠ Z ⎝ 2 m dz ⎠
and hence
1 ⎛ h2 d 2 X ⎞
⎜⎜ − + V ( x ) X ⎟⎟ = E x
X ⎝ 2m dx 2 ⎠
1 ⎛ h 2 d 2Y ⎞
⎜⎜ − 2
+ V ( y )Y ⎟⎟ = E y
Y ⎝ 2m dy ⎠
1 ⎛ h2 d 2Z ⎞
⎜⎜ − 2
+ V ( z ) Z ⎟⎟ = E z
Z ⎝ 2m dz ⎠
where the Ei is constants and Ex +Ey +Ez = E. The problem breaks into three one dimensional
infinite wells and we know that the solution to the one dimensional problem is
2 h 2π 2 2
X n x ( x) = sin( nxπx / L) with En x = nx where nx = 1, 2, 3, …
L 2mL2
2 h 2π 2 2
Yn y ( y ) = sin(n yπy / L) with En y = n y where ny = 1, 2, 3, …
L 2mL2
2 h 2π 2 2
Z nz ( z) = sin(nzπz / L) with En z = nz where nz = 1, 2, 3, …
L 2mL2
Thus,
8
ψ nxnynz ( x, y, z ) = X nx ( x)Yny ( y ) Z nz ( z ) = 3 sin( nxπx / L) sin( n yπy / L) sin( nzπz / L) ,
L
and
h 2π 2 2
E = E n x + En y + E n z = 2
(nx + n y2 + nz2 ) .
2mL
(b) (5 points) Call the energies E1, E2, E3, …, in order of increasing energy. Find E1, E2, E3, E4,
E5, and E6. Determine the degeneraries (i.e. the number of different states with the same
energy).

Department of Physics Page 1 of 9


PHY4604 Fall 2007 Problem Set 5 Solutions

Answer:
Energy nx ny nz Number of States
E1= 3h 2π 2 /(2mL2 ) 1 1 1 1
E2= 6h 2π 2 /(2mL2 ) 2 1 1
E2 1 2 1
E2 1 1 2 3
E3= 9h π /(2mL2 )
2 2 2 2 1
E3 2 1 2
E3 1 2 2 3
E4= 11h π /(2mL2 )
2 2 3 1 1
E4 1 3 1
E4 1 1 3 3
E5= 12h π /(2mL2 )
2 2 2 2 2 1
E6= 14h 2π 2 /(2mL2 ) 1 2 3
E6 1 3 2
E6 2 1 3
E6 2 3 1
E6 3 1 2
E6 3 2 1 6

(c) (5 points) What is the degeneracy of E14, and why is this case interesting?
Answer: The degeneracy of E14 is 4.
Solution: The combinations after E6 are E7(322), E8(411), E9(331), E10(421), E11(332), E12(422),
E13(431), and then E14(333 and 511). Simple combinatorics account for the degeneracies of 1 (nx
= ny = nz), 3 (two the same, one different), and 6 (all three different). But in the case of E14 there
is a numerical “accident”: 32 + 32 +32 = 27 and 52 +12 +12 = 27, so the degeneracy is greater than
combinatorial reasoning alone would suggest.

Problem 2 (20 points): A particle of mass m and E < 0 is placed in a finite spherical well
⎧− V r ≤ R
V (r ) = ⎨ 0
⎩ 0 r>R
(a) (10 points) Find the ground state by solving the radial equation with l = 0.
Solution: The radial equation is given by
h 2 d 2U (r ) l (l + 1)h 2
− + V ( r )U ( r ) = EU ( r ) where V ( r ) = V ( r ) + ,
2m dr 2
eff
2mr 2
eff

where U(r) = rR(r). In the region for r < L (region 1) with l = 0 we have
d 2U (r ) 2m( E + V0 ) h 2k 2
=− U (r ) = −k U (r ) where k = 2m( E + V0 ) / h and E =
2 2
− V0
dr 2 h2 2m
and U1 (r ) = A sin(kr ) + B cos(kr ) and R1 (r ) = A sin(kr ) / r + B cos(kr ) / r . The second term
diverges as r → 0 and hence B = 0. In the region for r > L (region 2) with l = 0 we have

Department of Physics Page 2 of 9


PHY4604 Fall 2007 Problem Set 5 Solutions

d 2U (r ) 2mE h 2κ 2
= − U ( r ) = κ 2
U ( r ) where κ = − 2 mE / h 2
and E = −
dr 2 h2 2m
−κr +κr
and U 2 (r ) = Ce + De . The second term diverges as r → ∞ and hence D = 0.
The boundary condition at r = R are
U1 ( R) = U 2 ( R) which implies (1) A sin(kR) = Ce−κR
dU1 dU 2
= which implies (2) kA cos(kR) = −κCe−κR
dr r =R dr r =R
We see by dividing (1)/(2) that
tan(kR) 1 κ
=− or − cot(kR) = .
k κ k
2 2
hk hκ2 2
2mV
Note that E = − V0 = − and hence κ 2 = 2 0 − k 2 .
2m 2m h
Let y = kR then
κ 2mV0 R 2 / h 2 − y 2
= = ( y0 / y) 2 − 1 where y0 = 2mV0 R 2 / h 2
k y
− cot( y) = ( y0 / y ) 2 − 1
h2 y2
and E = − V0 . We now let f(y) = -cot(y) and g ( y ) = ( y0 / y) 2 − 1 and plot.
2mR 2
π/2 π 3π/2 2π
5

4
y0 = 2π
3
f(y)
2 g(y)

0
0.0 1.6 3.1 4.7 6.3 7.9 y 9.4

-1

We see that g(y) = 0 (and then becomes imaginary) at y = y0 and hence there is no bound state
for y0 < π/2. If y0 ≥ π/2 then the ground state (i.e. lowest energy state) lies in the region
π/2 ≤ y ≤ π and the ground state energy lies in the range
h 2π 2 h 2π 2
− V0 ≤ E ≤ − V0 .
8mR 2 2mR 2
(b) (5 points) Show that there is no bound state if V0 R 2 < π 2h 2 / 8m.
Solution: We see from (a) that there is no bound state if
π
y0 = 2mV0 R 2 / h 2 < which implies V0 R 2 < π 2h 2 / 8m.
2
(c) (5 points) What are the l = 0 bound state energies for the case V0 R 2 = 2π 2h 2 / m ?
h 2π 2 h 2π 2
Answer: There are two bound states E1 ≈ −1.63 and E 2 ≈ −0.59
mR 2 mR 2

Department of Physics Page 3 of 9


PHY4604 Fall 2007 Problem Set 5 Solutions

Solution: In this case y0 = 2mV0 R 2 / h 2 = 2π and V0 = 2π 2h 2 /(mR 2 ) and there are two
solutions (see the plot). The ground state occurs when y ≈ 2.698 with energy
h 2π 2 h 2π 2 2h 2π 2 h 2π 2
E1 ≈ 0.37 − V0 = 0.37 − = − 1 .63 .
mR 2 mR 2 mR 2 mR 2
For the 1st excited state I get y ≈ 5.2845 with energy
h 2π 2 2h 2π 2 h 2π 2
E2 ≈ 1.41 − = −0.59 .
mR 2 mR 2 mR 2

Problem 3 (20 points): What is the most probable value of r, in the ground state of hydrogen?
Express your answer in terms of the Bohr radius r0, where
h2 ⎛ h ⎞⎛ hc ⎞ D e Ke 2 h
r0 = = ⎜
⎜ ⎟
⎟⎜ 2⎟ = with α = and D e = .
⎝ me c ⎠⎝ Ke ⎠ α
2
me Ke hc me c
Answer: The most probable value is r0
Solution: The probability of finding the electron between r and r+dr is
2π π
4
ρ (r ) = ∫ ∫ψ 100

(r ,θ ,φ )ψ 100 (r ,θ ,φ )r 2 sin θdθdφ = 3 r 2e − 2 r / r0
0 0
r0
and the most probable value occurs when
dρ ( r ) 4 ⎛ r 2 − 2 r / r0 − 2 r / r0 ⎞ 8re −2 r / r0 ⎛ r ⎞

= 3 ⎜− 2 e + 2re ⎟
⎟= ⎜⎜ − + 1⎟⎟ = 0
dr r0 ⎝ r0 ⎠ r03 ⎝ r0 ⎠
which implies that rmax = r0.

Problem 4 (20 points): The (time independent) momentum space wave function in three
dimensions is defined by
r 1 r r r
3/ 2 ∫
φ ( p) = e − i ( p ⋅r ) / hψ (r )d 3r .
(2πh)
(a) (10 points) Find the momentum space wave function for the ground state of hydrogen. Use
r r r
spherical coordinates and set the polar axis along the direction of p so that p ⋅ r = pr cosθ .
3/ 2
r 1 ⎛ 2r ⎞ 1
Answer: φ ( p ) = ⎜ 0 ⎟
π ⎝ h ⎠ (1 + (r0 p / h) 2 ) 2
Solution: For the ground state of hydrogen we have
∞ 2π π
r 1 r r
3/ 2 ∫ ∫ ∫
φ ( p) = e − i ( p⋅r ) / h
ψ 100 (r ,θ ,φ )r 2 sin θdθdφ
(2πh) 0 0 0
∞ 2π π
1 1
∫ ∫∫e
− i ( pr cos θ ) / h − r / r0
= e r 2 sin θdθdφ
(2πh ) 3/ 2
πr03 0 0 0
∞π
1 2π
∫∫e
− i ( pr cos θ ) / h − r / r0
e r 2 sin θdθ
(2πh ) 3/ 2
πr03 0 0

where I did the integral over φ. Now let y = cosθ and

Department of Physics Page 4 of 9


PHY4604 Fall 2007 Problem Set 5 Solutions

π 1

∫e
− i ( pr cos θ ) / h
sin θdθ = ∫ e − i ( prξ ) / h dξ =
pr
(
ih − ipr / h
e )
− e + ipr / h =
2h
pr
sin( pr / h ) and
0 −1

r 1 4πh
φ ( p) =
(2πh) p πr0 0
3/ 2 3 ∫ e − r / r0 r sin( pr / h)dr


1 2h − r / r0
p ∫0
= e r sin( pr / h)dr
π 2 (hr0 ) 3/ 2

Now
1 ⎡ − r / r0 ipr / h ⎤
∞ ∞ ∞

∫e r sin( pr / h)dr = ⎢ ∫ e re dr − ∫ e − r / r0 re − ipr / h dr ⎥


− r / r0

0
2i ⎣ 0 0 ⎦
1 ⎡ ( −1 / r0 + ip / h ) r ⎤
∞ ∞
= ⎢ ∫ re dr − ∫ re − (1 / r0 + ip / h ) r dr ⎥
2i ⎣ 0 0 ⎦

1⎡ ⎛ r 1 ⎞⎤
= ⎢e ( −1 / r0 + ip / h ) r ⎜⎜ − ⎟
2 ⎟⎥
2i ⎣ ⎝ − 1 / r0 + ip / h (−1 / r0 + ip / h) ⎠⎦ 0

1⎡ ⎛ r 1 ⎞⎤
− ⎢e − (1 / r0 + ip / h ) r ⎜⎜ − ⎟
2 ⎟⎥
2i ⎣ ⎝ − (1 / r0 + ip / h) (1 / r0 + ip / h ) ⎠⎦ 0
1⎡ 1 1 ⎤ 1 4ip /(r0h)
= ⎢ − 2⎥
=
2i ⎣ (1 / r0 − ip / h ) 2
(1 / r0 + ip / h) ⎦ 2i ((1 / r0 ) 2 + ( p / h ) 2 ) 2
4r03 p /(h )
=
(1 + (r0 p / h ) 2 ) 2
and hence
3/ 2
r 1 2h 4r03 p /(h) 1 ⎛ 2r ⎞ 1
φ ( p) = = ⎜ 0⎟ .
π 2 (hr0 ) 3/ 2
p (1 + (r0 p / h) ) 2 2
π ⎝ h ⎠ (1 + (r0 p / h) 2 ) 2
r
(b) (5 points) Show that φ ( p ) is normalized.
Solution: We see that
∞ 3∞
r 2 3 4π ⎛ 2r0 ⎞ p2
∫ φ = π ∫0 φ =
π 2 ⎝ h ⎠ ∫0 (1 + (r0 p / h ) 2 ) 4
2 2
| ( p ) | d p 4 | | p dp ⎜ ⎟ dp .

From the integral tables I see that


x2 x x x 1
∫ (a 2 + x 2 )4 dx = − 6(a 2 + x 2 )3 + 24a 2 (a 2 + x 2 )2 + 16a 4 (a 2 + x 2 ) + 16a5 tan ( x / a)
−1

and hence

x2 1 π π
∫0 (a 2 + x 2 )4 dx = 16a5 2 = 32a5 .
Thus,
∞ ∞
p2 1 p2 1 π 1 π
∫0 (1 + (r0 p / h)2 ) 4 dp = 8 ∫
(r0 / h ) 0 ((h / r0 ) + p )
2 2 4
dp = 8
(r0 / h ) 32(h / r0 ) 5
= 3
(r0 / h ) 32

Department of Physics Page 5 of 9


PHY4604 Fall 2007 Problem Set 5 Solutions

and
3∞ 3
r 2 3 32 ⎛ r0 ⎞ p2 32 ⎛ r ⎞ 1 π
∫ | φ ( p ) | d p = ⎜ ⎟ ∫
π ⎝ h ⎠ 0 (1 + (r0 p / h) )
2 4
dp = ⎜ 0 ⎟
π ⎝ h ⎠ (r0 / h) 32
3
= 1.
r
(b) (5 points) Use φ ( p) to calculate <p2>, in the ground state of hydrogen.
h2
Answer: < p 2 >=
r02
Solution:
∞ 3∞
r 32 ⎛ r ⎞ p4
< p >= ∫ p | φ ( p) |2 d 3 p = 4π ∫ | φ |2 p 4 dp = ⎜ 0 ⎟ ∫
2 2
dp .
0
π ⎝ h ⎠ 0 (1 + (r0 p / h) 2 ) 4
From the integral tables I see that
x4 a2 x 7x x 1
∫ (a 2 + x 2 )4 dx = 6(a 2 + x 2 )3 − 24(a 2 + x 2 )2 + 16a 2 (a 2 + x 2 ) + 16a3 tan ( x / a)
−1

and hence

x4 1 π π
∫0 (a 2 + x 2 )4 dx = 16a3 2 = 32a3 .
Thus,
3∞ 3 ∞
32 ⎛ r0 ⎞ p4 32 ⎛ r0 ⎞ 1 p4
< p >= ⎜ ⎟ ∫
π ⎝ h ⎠ (r0 / h)8 ∫0 ((h / r0 ) 2 + p 2 ) 4
2
dp = ⎜ ⎟ dp
π ⎝ h ⎠ 0 (1 + (r0 p / h) 2 ) 4
3
32 ⎛ r ⎞ 1 π h2
= ⎜ 0⎟ =
π ⎝ h ⎠ (r0 / h)8 32(h / r0 )3 r02

Problem 5 (20 points): Angular momentum is the vector operator given by


xˆ yˆ zˆ
r r r
Lop = r × p op = x y z
( p x ) op ( p y ) op ( p z ) op
Hence,
( Lx ) op = y ( p z ) op − z ( p y ) op ( Ly )op = z ( px )op − x( pz )op ( Lz ) op = x( p y ) op − y ( p x ) op .
(a) (5 points) Starting with the canonical commutations for position and momentum, work out
the following commutators:
[ Lz , x] = ihy , [ Lz , y ] = −ihx , [ Lz , z ] = 0 ,
[ Lz , px ] = ihp y , [ Lz , p y ] = −ihpx , [ Lz , p z ] = 0 .
Solution:
[ Lz , x] = [ xp y − ypx , x] = [ xp y , x] − [ ypx , x] = −[ ypx , x] = − y[ px , x] = ihy
[ Lz , y ] = [ xp y − ypx , y ] = [ xp y , y ] − [ ypx , y ] = [ xp y , y ] = x[ p y , y ] = −ihx
[ Lz , z ] = [ xp y − ypx , z ] = [ xp y , z ] − [ ypx , z ] = 0
[ Lz , px ] = [ xp y − ypx , px ] = [ xp y , px ] − [ ypx , px ] = [ xp y , px ] = [ x, px ] p y = ihp y

Department of Physics Page 6 of 9


PHY4604 Fall 2007 Problem Set 5 Solutions

[ Lz , p y ] = [ xp y − ypx , p y ] = [ xp y , p y ] − [ ypx , p y ] = −[ ypx , p y ] = −[ y, p y ] p x = −ihp x


[ Lz , pz ] = [ xp y − ypx , pz ] = [ xp y , pz ] − [ ypx , pz ] = 0
(b) (5 points) Use the results of (a) to prove that [ Lz , Lx ] = ihLy .
Solution:
[ Lz , Lx ] = [ Lz , ypz − zp y ] = [ Lz , ypz ] − [ Lz , zp y ] =
y[ Lz , pz ] + [ Lz , y ] pz − z[ Lz , p y ] − [ Lz , z ] p y =
= 0 − ihxpz + ihzpx − 0 = ihLy
(c) (5 points) Evaluate the commutators [ Lz , r 2 ] and [ Lz , p 2 ] , where r 2 = x 2 + y 2 + z 2 and
p 2 = p x2 + p y2 + pz2 .
Answer: [ Lz , r 2 ] = 0 and [ Lz , p 2 ] = 0
Solution:
[ Lz , r 2 ] = [ Lz , x 2 + y 2 + z 2 ] = [ Lz , x 2 ] + [ Lz , y 2 ] = x[ Lz , x] + [ Lz , x]x + y[ Lz , y ] + [ Lz , y ] y
= ihxy + ihyx − ihyx − ihxy = 0
[ Lz , p 2 ] = [ Lz , px2 + p y2 + pz2 ] = [ Lz , px2 ] + [ Lz , p y2 ] = px [ Lz , px ] + [ Lz , px ] px + p y [ Lz , p y ] + [ Lz , p y ] p y
= ihpx p y + ihp y px − ihp y px − ihpx p y = 0
Note that one can show (in a similar way) that [ Li , r 2 ] = [ Li , p 2 ] = 0 for i = x, y, z.
p2 r
(d) (5 points) Show that the Hamiltonian H = + V commutes with all three components of L
2m
provided that V depends only on r = x 2 + y 2 + z 2 .
Solution:
[ Li , H ] = [ Li , p 2 / 2m + V (r )] = [ Li ,V (r )]
where i = x, y, z and where I used the results from (c).
Now
⎛ ∂ ∂ ⎞
[ Lx ,V (r )] = ih⎜⎜ sin φ + cot θ cos φ ⎟V (r ) = 0
⎝ ∂θ ∂φ ⎟⎠
⎛ ∂ ∂ ⎞
[ Ly ,V (r )] = ih⎜⎜ − cos φ + cot θ sin φ ⎟V (r ) = 0
⎝ ∂θ ∂φ ⎟⎠

[ Lz ,V (r )] = −ih V (r ) = 0
∂φ
Thus,
[ Li , H ] = 0 .

Problem 6 (20 points): The three components of the vector hermitian operator
r
J = J x xˆ + J y yˆ + J z zˆ obey the “lie algebra”: [Ji,Jj] = iεijkJk, where J1 = Jx, J2 = Jy, and J3 = Jz.
(a) (3 points) Prove that [J2,Jk] = 0 (k=1,2,3), where J2 = Jx2 + Jy2 + Jz2.
Solution:

Department of Physics Page 7 of 9


PHY4604 Fall 2007 Problem Set 5 Solutions

[J2,Jx] = [Jx2 + Jy2 + Jz2,Jx] = [Jy2 + Jz2,Jx] = [Jy2,Jx] + [Jz2,Jx]


= Jy [Jy,Jx] + [Jy,Jx] Jy + Jz [Jz,Jx] + [Jz,Jx] Jz
= -i Jy Jz –i Jz Jy +iJz Jy +i Jy Jz = 0
Similarly [J ,Jy] = 0 and [J2,Jz] = 0.
2

(b) (5 points) Define |jm> to be an eigenket of J2 and Jz with eigenvalues given by


J2|jm> = j(j+1)|jm> and Jz|jm> = m|jm> with <j'm'|jm> = δj'jδm'm. Prove that
J±|jm> = c±|jm±1>
where J± = Jx±iJy.
Solution: First we must show that [J2,J±] = 0 and [ J z , J ± ] = ± J ± as follows:
[ J 2 , J ± ] = [ J 2 , J x ± iJ y ] = [ J 2 , J x ] ± i[ J 2 , J y ] = 0 and
[ J z , J ± ] = [ J z , J x ± iJ y ] = [ J z , J x ] ± i[ J z , J y ] = iJ y ± i (−iJ x ) = ± ( J x ± iJ y ) = ± J ± .
Now we look at the state (J±|jm>) and we see that
J 2 ( J ± | jm >) = J ± J 2 | jm >= j ( j + 1)( J ± | jm >) .
Thus the state (J±|jm>) is an eigenket of J2 with eigenvalue j(j+1). Now look at
J z ( J ± | jm >) = ( J ± J z ± J ± ) | jm >= ( J ± m ± J ± ) | jm >= (m ± 1)( J ± | jm >) .
Thus the state (J±|jm>) is an eigenket of Jz with eigenvalue m±1 and hence J±|jm> = c±|jm±1>,
where c± are constants that can be determined by normalizing the eigenkets.
(c) (5 points) Show that c± = j ( j + 1) − m(m ± 1) .
Solution: First we must show that J+J-= J2- Jz(Jz-1) and J-J+= J2- Jz(Jz+1) as follows
J m J ± = ( J x m iJ y )( J x ± iJ y ) = J x2 + J y2 ± i[ J x , J y ] = J x2 + J y2 m J z = J 2 − J z2 m J z = J 2 − J z ( J z ± 1) .
Now we normalize eigenkets and look at the norm of the state J±|jm> = c±|jm±1> as follows
< jm | J m J ± | jm >=< jm | J 2 − J z ( J z ± 1) | jm >= ( j ( j + 1) − m(m ± 1)) < jm | jm >
=| c± |2 < jm ± 1 | jm ± 1 >
Since the eigenkets are normalized so that <j'm'|jm> = δj'jδm'm we see that
c± = j ( j + 1) − m(m ± 1) .
(d) (3 points) Prove that m2 ≤ j(j+1).
Solution: We see that
0 ≤ <jm|(Jx)2|jm> + <jm|(Jy)2|jm> = <jm|J2 -(Jz)2|jm> = j(j+1)-m2
and hence j(j+1)-m2 ≥ 0 and j(j+1) ≥ m2.
(e) (4 points) Prove that mmax = j and mmin = -j.
Solution: We see that
| c+ |2 = j ( j + 1) − m(m + 1) ≥ 0
and thus mmax = j. Also,
| c− |2 = j ( j + 1) − m(m − 1) ≥ 0 .
If I let m= -n then I see that
| c− |2 = j ( j + 1) − (−n)(−n − 1) = j ( j + 1) − n(n + 1) ≥ 0
and hence nmax = j which means that mmin = -j.

Problem 7 (10 points): Two particles of mass m are attached to the ends of a massless rigid rod
of length a. The system is free to rotate in three dimensions about the center (but the center point
itself is fixed).

Department of Physics Page 8 of 9


PHY4604 Fall 2007 Problem Set 5 Solutions

(a) (5 points) Show that the allowed energies of this rigid rotator are
h 2 n(n + 1)
En = where n = 0, 1, 2, …
ma 2
Solution: We first have to construct the Hamiltonian. The energy is
⎛1 ⎞ (amv) 2 L2
E = 2⎜ mv 2 ⎟ = mv 2 = = ,
⎝2 ⎠ ma 2 ma 2
where L is the total angular momentum of the system. Thus, the Hamiltonian is
L2
H op = op2 and the we are looking for the solutions of the eigenvalue equation
ma
L2op
ψ (θ , φ ) = Eψ (θ , φ ) or L2opψ (θ ,φ ) = ma 2 Eψ (θ ,φ ) .
ma 2
We know that Ylm are the eigenfunctions of the (L2)op with eigenvalues l (l + 1)h 2 as follows
L2opYlm (θ ,φ ) = l (l + 1)h 2Ylm (θ ,φ ) = ma 2 EYlm (θ ,φ ) ,
and hence
h l (l + 1)
2
El = where l = 0, 1, 2, …
ma 2
(b) (5 points) What are the normalized eigenfunctions for this system? What is the degeneracy
of the nth energy level?
Answer: The normalized eigenfunction are Ylm(θ,φ) and the nth level has (2n+1) states
corresponding to the (2l+1) m values that occur for each l.

Department of Physics Page 9 of 9

You might also like